Autor Tema: Pistas para un teorema del punto fijo

0 Usuarios y 1 Visitante están viendo este tema.

16 Febrero, 2022, 05:16 am
Respuesta #20

Marcos Castillo

  • $$\Large \color{#9c57a6}\pi\,\pi\,\pi\,\pi\,\pi\,\pi$$
  • Mensajes: 2,515
  • País: es
  • Karma: +1/-0
  • Sexo: Masculino
Hola, Luis, estimado Rincón

Este gráfico de Geogebra para un caso concreto (\( x=e^{-x} \)) me ha ayudado un poco.

Voy a repetir el enunciado del teorema (la primera parte y su comprobación), luego la imagen, y finalmente dos dudas:

Citar

El siguiente teorema garantiza para qué funciones el método de iteración del punto fijo funciona:

TEOREMA 8 Un teorema del punto fijo

Supongamos que \( f \) está definida en un intervalo \( I=[a,b] \) y cumple las dos siguientes condiciones:

(i) \( f(x) \) pertenece a \( I \) siempre que \( x \)  pertenezca a \( I \).
(ii) Existe una constante \( K \), con \( 0<K<1 \), tal que para todos \( u \) y \( v \) pertenecientes a \( I \),

\( |f(u)-f(v)|\leq K|u-v| \)

Entonces \( f \) tiene un punto fijo \( r \) en \( I \), es decir, \( f(r)=r \), y empezando con cualquier número \( x_0 \) en \( I \), las iteraciones

\( x_1=f(x_0),\qquad x_2=f(x_1),... \)

convergen a \( r \).

Prueba (voy a por la primera de las dos)

La condición (ii) del Teorema 8 implica que \( f \) es continua en \( I=[a,b] \). Utilice la condición (i) para demostrar que \( f \) tiene un punto fijo \( r \) en \( I \).
Sugerencia: Aplique el Teorema del Valor Intermedio a \( g(x)=f(x)-x \) en \( [a,b] \)


Conclusión que he sacado. El Teorema del Valor Intermedio me ha servido para encontrar en el gráfico algo parecido a un punto fijo: \( f(r)\approx{r} \)



Dudas

Estoy seguro del \( r \) que planteo para la ecuación \( x=e^{-x} \); proviene de un tutorial que parece serio.
¿Por qué \( f(r) \) sólo se aproxima unos pocos decimales, y luego difiere?
¿Cómo es que me estoy apoyando en la condicción (i)? ¿Porque \( [0,1]\subseteq{[0,1]} \)? ¿Que trascendencia tiene esto?

¡Un saludo!

16 Febrero, 2022, 06:53 am
Respuesta #21

Juan Pablo Sancho

  • Moderador Global
  • Mensajes: 6,261
  • País: es
  • Karma: +0/-0
  • Sexo: Masculino
Si tienes \(  f(I) \subset I  \) queda que \(  f(a) \geq a  \) y \( f(b) \leq b  \) luego:
\( g(x) = f(x)-x  \) tenemos \( g(a) \geq 0  \) y \( g(b) \leq 0  \) luego por Bolzano existe \( \delta \in [a,b]  \) con \( g(\delta) = \delta  \)   

Editado
Lo siguiente está mal.
Luego \( g(x) = e^{-x} - x  \) tenemos \( g'(x) = -e^{-x} - 1 \) nos queda \( |g'(x)| > 1  \) el punto será repelente.

16 Febrero, 2022, 10:35 pm
Respuesta #22

Marcos Castillo

  • $$\Large \color{#9c57a6}\pi\,\pi\,\pi\,\pi\,\pi\,\pi$$
  • Mensajes: 2,515
  • País: es
  • Karma: +1/-0
  • Sexo: Masculino
Estimado Rincón:
Perdón, acabo de leer mi último mensaje, y os ruego que sólo os quedéis con... Con nada. Es más, el anterior que publiqué no le va a la zaga. Lo siento, de verdad.  :'(
No sé qué más decir. Bueno, lo que voy a hacer es imprimir el hilo, y estudiarlo.
Un saludo

25 Febrero, 2022, 07:09 am
Respuesta #23

Marcos Castillo

  • $$\Large \color{#9c57a6}\pi\,\pi\,\pi\,\pi\,\pi\,\pi$$
  • Mensajes: 2,515
  • País: es
  • Karma: +1/-0
  • Sexo: Masculino
Si tienes \(  f(I) \subset I  \) queda que \(  f(a) \geq a  \) y \( f(b) \leq b  \) luego:
\( g(x) = f(x)-x  \) tenemos \( g(a) \geq 0  \) y \( g(b) \leq 0  \) luego por Bolzano existe \( \delta \in [a,b]  \) con \( g(\delta) = \delta  \)   
Genial
Luego \( g(x) = e^{-x} - x  \) tenemos \( g'(x) = -e^{-x} - 1 \) nos queda \( |g'(x)| > 1  \) el punto será repelente.

https://foro.rinconmatematico.com/index.php?topic=119482.0

!Hola primero de todo!

Creo que \( g(x) \) es en este ejercicio \( e^{-x} \)

Juan Pablo, el_manco, geómetracac, Rincón, me sentía tan avergonzado por mis anteriores intervenciones, que he estado estudiando el tema con otro foro, Physics Forums. Y lo que he hecho es en realidad todo lo que me aconsejaba Luis. Creo que se ha solucionado, pero tengo dudas, que en mi próximo mensaje os cuento.

¡Un saludo cordial!

25 Febrero, 2022, 09:51 am
Respuesta #24

Juan Pablo Sancho

  • Moderador Global
  • Mensajes: 6,261
  • País: es
  • Karma: +0/-0
  • Sexo: Masculino
Se me fue la pinza con la función.

27 Febrero, 2022, 08:31 am
Respuesta #25

Marcos Castillo

  • $$\Large \color{#9c57a6}\pi\,\pi\,\pi\,\pi\,\pi\,\pi$$
  • Mensajes: 2,515
  • País: es
  • Karma: +1/-0
  • Sexo: Masculino
Hola, estimado Rincón, creía haberlo solucionado casi, pero redactando el mensaje me han surgido dos dudas. Primero repito el enunciado del Teorema 8, y luego las soluciones y las dudas 1 y 2.


La iteración del método del punto fijo no funciona siempre. Funciona para una clase particular de funciones. El siguiente teorema garantiza que la iteración del método del punto fijo funciona para ciertas funciones:

TEOREMA 8 Un teorema del punto fijo

Supongamos que \( f \) está definida en un intervalo \( I=[a,b] \) y cumple las dos siguientes condiciones:

(i) \( f(x) \) pertenece a \( I \) siempre que \( x \)  pertenezca a \( I \).
(ii) Existe una constante \( K \), con \( 0<K<1 \), tal que para todos \( u \) y \( v \) pertenecientes a \( I \),

\( |f(u)-f(v)|\leq K|u-v| \)

Entonces \( f \) tiene un punto fijo \( r \) en \( I \), es decir, \( f(r)=r \), y empezando con cualquier número \( x_0 \) en \( I \), las iteraciones

\( x_1=f(x_0),\qquad x_2=f(x_1),... \)


convergen a \( r \).

Prueba

(a) La condición (ii) del Teorema 8 implica que \( f \) es continua en \( I=[a,b] \). Utilice la condición (i) para demostrar que \( f \) tiene un punto fijo \( r \) en \( I \).
Sugerencia: Aplique el Teorema del Valor Intermedio a \( g(x)=f(x)-x \) en \( [a,b] \)

(b) Utilice la condición (ii) del Teorema 8 e inducción matemática para demostrar que

\( |x_n-r|\leq K^{n}|x_0-r| \)

Como \( 0<K<1 \), sabemos que \( K^{n}\rightarrow{0} \) cuando \( n\rightarrow{\infty} \). Esto demuestra que \( \lim_{n\rightarrow{\infty}}{x_n}=r \).

(a)

1) Justificar que \( g(x) \) es continua en \( [a,b] \). Ya probado.

2) Si \( g(a)=0 \) o \( g(b)=0 \), comprobar que ya tenemos punto fijo. Fácil

3) De otra forma, probar que \( g(a) \) y \( g(b) \) tienen diferentes signos. Una raíz de \( g(x) \) cumple que \( f(c)-c=0 \)

Si \( g(a)\neq 0 \), y \( g(b)\neq 0 \)... Se observa que si \( f(I)\subseteq{I} \), concluímos que \( f(a)\geq a \) y \( f(b)\leq b \). Aplicado esto a \( g(x)=f(x)-x \), \( g(a)\geq 0 \) y \( g(b)\leq 0 \). Teorema de Bolzano: existe un \( \delta\in{[a,b]} \) tal que \( g(\delta)=\delta \)

Esta es la solución que me han proporcionado. Pero, ¿por qué si \( f(I)\subseteq{I} \), concluímos que \( f(a)\geq a \) y \( f(b)\leq b \)?

Cita de: Physics Forums
\( I \) está definido como \( [a,b] \), es decir, como \( \{x\in\Bbb{R}\;:\;a\leq x\leq b\} \). De esta forma, decir que \( f(a) \) está en \( I \) es otra forma de decir que \( f(a) \) es real y \( f(a)\geq{a} \) y \( f(a)\leq{b} \). Similarmente para \( f(b) \)

Duda 1: ¿Por qué \( f(I)\subseteq{I} \) implica \( f(a)\geq{a} \) y \( f(a)\leq{b} \). Similarmente por qué \( f(b)\leq{a} \) y \( f(b)\geq{b} \) (no estoy seguro de estas dos últimas desigualdades)

(b)

Utilice la condición (ii) del Teorema 8 e inducción matemática para demostrar que

\( |x_n-r|\leq K^{n}|x_0-r| \)

Como \( 0<K<1 \), sabemos que \( K^{n}\rightarrow{0} \) cuando \( n\rightarrow{\infty} \). Esto demuestra que \( \lim_{n\rightarrow{\infty}}{x_n}=r \).


Debemos probar que

\( |x_n-r|\leq K^n |x_0-r| \)

1-Observemos para \( n=1 \) y \( n=2 \) (probar base inductiva)

\( |x_1-r|=|f(x_0)-f(r)|\leq K|x_0-r| \)

\( |x_2-r|=|f(x_1)-f(r)|\leq K|x_1-r|\leq\cdot K\cdot |x_0-r|=K^2|x_0-r| \)

2-Asumamos \( n=s \) (plantear la hipótesis)

\( |x_s-r|\leq K^s |x_0-r| \)

3- Demostrar igualdad \( n=s+1 \), para \( s\in{\Bbb{K}} \)

\( |x_{s+1}-r|=|f(x_s)-f(r)|\leq K^{s+1}|x_0-r| \)

Pero \( \begin{cases}K\in{\Bbb{R}}\quad{0<K<1}\\s\in{\Bbb{N}}\end{cases}\Rightarrow{K^{s+1}<K^s} \)

\( |x_{s+1}-r|=|f(x_s)-f(r)|\leq K|x_s - r|\leq K \cdot K^s|x_0-r|= K^{s+1}|x_0-r| \)

Duda 2: me lío. Digo "Pero" hace dos líneas y escribo \( < \):sin embargo en la línea siguiente son \( \leq{} \)

¡Un saludo!

27 Febrero, 2022, 03:20 pm
Respuesta #26

Juan Pablo Sancho

  • Moderador Global
  • Mensajes: 6,261
  • País: es
  • Karma: +0/-0
  • Sexo: Masculino
Si \( f(I) \subset I = [a,b]  \) tenemos que \( f(a) \in [a,b]  \) luego \( a \leq f(a) \leq b  \) lo mismo con \( f(b) \).
Por lo anterior , si \( f(b) \leq a  \) sólo ocurre si \( f(b)=a  \) será \(  a \leq f(b) \leq b  \) por lo anterior , por ser \( f(b) \in [a,b]  \)

Para la inducción Sea \( x_o \in [a,b]  \), luego tenemos que:
\( |x_1 - r| = |f(x_0) - f(r)| \leq k \cdot |x_0 - r|  \) es cierto.
\( |x_2 - r| = |f(x_1) - f(r)| \leq k \cdot |x_1 - r| \leq  k \cdot k \cdot |x_0 - r| =  k^2 \cdot |x_0 - r|  \).

Lo suponemos cierto hasta \( n \) que \( |x_n - r| \leq k^n \cdot |x_0 - r|  \).
Luego para \( n+1 \).
\( |x_{n+1} - r|  = |f(x_n) - f(r)| \leq k \cdot |x_n - r| \leq k \cdot k^n \cdot |x_0 - r| = k^{n+1} \cdot |x_0 - r|  \)

\( \leq  \) si lo sustituyes por \( < \) , que pasa con el caso \( x_0 = r \), quedaría \( 0<0 \).

28 Febrero, 2022, 12:52 pm
Respuesta #27

Marcos Castillo

  • $$\Large \color{#9c57a6}\pi\,\pi\,\pi\,\pi\,\pi\,\pi$$
  • Mensajes: 2,515
  • País: es
  • Karma: +1/-0
  • Sexo: Masculino
Si \( f(I) \subset I = [a,b]  \) tenemos que \( f(a) \in [a,b]  \) luego \( a \leq f(a) \leq b  \) lo mismo con \( f(b) \).
Por lo anterior , si \( f(b) \leq a  \) sólo ocurre si \( f(b)=a  \) será \(  a \leq f(b) \leq b  \) por lo anterior , por ser \( f(b) \in [a,b]  \)
¡Perfecto! Ya me ha costado
Para la inducción Sea \( x_o \in [a,b]  \), luego tenemos que:
\( |x_1 - r| = |f(x_0) - f(r)| \leq k \cdot |x_0 - r|  \) es cierto.
\( |x_2 - r| = |f(x_1) - f(r)| \leq k \cdot |x_1 - r| \leq  k \cdot k \cdot |x_0 - r| =  k^2 \cdot |x_0 - r|  \).

Lo suponemos cierto hasta \( n \) que \( |x_n - r| \leq k^n \cdot |x_0 - r|  \).
Luego para \( n+1 \).
\( |x_{n+1} - r|  = |f(x_n) - f(r)| \leq k \cdot |x_n - r| \leq k \cdot k^n \cdot |x_0 - r| = k^{n+1} \cdot |x_0 - r|  \)

\( \leq  \) si lo sustituyes por \( < \) , que pasa con el caso \( x_0 = r \), quedaría \( 0<0 \).

Hola

Para el caso en que \( x_0=r \), solamente hay una iteración, ¿no?: \( |x_1-r|=|f(x_0)-f(r)|\leq K |x_0-r|\Rightarrow{x_1=f(x_0)=f(r)}\Rightarrow{0\leq 0} \)

¡Un saludo!

28 Febrero, 2022, 12:56 pm
Respuesta #28

Juan Pablo Sancho

  • Moderador Global
  • Mensajes: 6,261
  • País: es
  • Karma: +0/-0
  • Sexo: Masculino
Si, solo hay una \( x_n = r  \) para todo \( n \in \mathbb{N} \cup \{0\}   \).

28 Febrero, 2022, 02:51 pm
Respuesta #29

Marcos Castillo

  • $$\Large \color{#9c57a6}\pi\,\pi\,\pi\,\pi\,\pi\,\pi$$
  • Mensajes: 2,515
  • País: es
  • Karma: +1/-0
  • Sexo: Masculino
¡Entendido! Sigo adelante.
¡Mil gracias, Rincón!